subject
Mathematics, 09.12.2020 02:50 Wxaieyu

I need help on this question


I need help on this question

ansver
Answers: 1

Another question on Mathematics

question
Mathematics, 21.06.2019 19:00
1) in triangle the, what is the measure of angle t (in degrees)? 2) in triangle bat, the measure of angle b is 66∘, and the measure of angle t is 77∘.what is the measure of the exterior angle at a (in degrees)?
Answers: 1
question
Mathematics, 21.06.2019 20:50
Determine the common ratio and find the next three terms of the geometric sequence. 649-14-02-00-00_files/i0100000, 649-14-02-00-00_files/i0100001, 2, a. 3; 649-14-02-00-00_files/i0100002, 649-14-02-00-00_files/i0100003, 649-14-02-00-00_files/i0100004. c. 649-14-02-00-00_files/i0100005; 649-14-02-00-00_files/i0100006, 649-14-02-00-00_files/i0100007, 649-14-02-00-00_files/i0100008. b. 3; 6, 18, 54 d. 649-14-02-00-00_files/i0100009. 6, 18, 54 select the best answer from the choices provided a b c d
Answers: 1
question
Mathematics, 21.06.2019 22:10
If p(a) = 0.70 and p(b) = 0.20, then a and b are independent events if
Answers: 3
question
Mathematics, 21.06.2019 23:00
The equation shown below represents function f. f(x)= -2x+5 the graph shown below represents function g. which of the following statements is true? a. over the interval [2, 4], the average rate of change of f is the same as the average rate of change of g. the y-intercept of function f is less than the y-intercept of function g. b. over the interval [2, 4], the average rate of change of f is greater than the average rate of change of g. the y-intercept of function f is greater than the y-intercept of function g. c. over the interval [2, 4], the average rate of change of f is the same as the average rate of change of g. the y-intercept of function f is greater than the y-intercept of function g. d. over the interval [2, 4], the average rate of change of f is less than the average rate of change of g. the y-intercept of function f is the same as the y-intercept of function g.
Answers: 1
You know the right answer?
I need help on this question
...
Questions
Questions on the website: 13722361